Đến nội dung

Cantho2015 nội dung

Có 47 mục bởi Cantho2015 (Tìm giới hạn từ 20-04-2020)



Sắp theo                Sắp xếp  

#640978 Topic tổng hợp các bài toán về phương trình nghiệm nguyên.

Đã gửi bởi Cantho2015 on 18-06-2016 - 04:14 trong Số học

Tìm $x , y \geq 0$ biết $\left ( xy - 7 \right )^{2} = x^{2} + y^{2}$

 

Giải:

$(xy-7)^2=x^2+y^2 \Leftrightarrow (xy-7)^2+2(xy-7)+1=x^2+y^2+2xy-13$

$(xy-6)^2-(x+y)^2=-13=13.(-1)$

Vì $x,y \geq 0$ nên $xy-6+(x+y) \geq xy-6-(x+y)$

$ \Rightarrow$

$$\begin{cases} xy-6+(x+y)=13\\xy-6-(x+y)=-1 \end{cases}$$

$ \Rightarrow$

$$\begin{cases} xy=6\\x+y=7 \end{cases}$$

$\Rightarrow$ $(x;y)=(3;4),(4;3)$




#641347 Topic tổng hợp các bài toán về phương trình nghiệm nguyên.

Đã gửi bởi Cantho2015 on 20-06-2016 - 04:57 trong Số học

mọi người xem giúp mình bài này với ..giair phương trình với x,y nguyên: $x^y=y^x$

Mình nghĩ là x,y phải nguyên dương

Nếu x,y nguyên dương, không mất tính tổng quát, giả sử $x>y$ và $(x;y)=d (d>0)$

$\Rightarrow x=dy$

pt $\Leftrightarrow (dy)^y-(y^d)^y=[dy-(y^d)]A (A>0)$

$\Rightarrow dy=y^d$

Với $d=1$ suy ra $x=y$

Với $d>1$, dễ thấy $y^d>dy$, vậy pt vô nghiệm




#643358 Topic tổng hợp các bài toán về phương trình nghiệm nguyên.

Đã gửi bởi Cantho2015 on 02-07-2016 - 22:00 trong Số học

Mình nghĩ là x,y phải nguyên dương

Nếu x,y nguyên dương, không mất tính tổng quát, giả sử $x>y$ và $(x;y)=d (d>0)$

$\Rightarrow x=dy$

pt $\Leftrightarrow (dy)^y-(y^d)^y=[dy-(y^d)]A (A>0)$

$\Rightarrow dy=y^d$

Với $d=1$ suy ra $x=y$

Với $d>1$, dễ thấy $y^d>dy$, vậy pt vô nghiệm

Bài giải thiếu nghiệm với phần chứng minh $x=dy$ mình bổ sung:

$gcd(x,y)=1 \Rightarrow x=dx_1, y=dy_1$ với $ gcd(x_1,y_1)=1$ và $x_1 > y_1$ ( vì $x>y$, $x=y$ thì xét riêng)

$x^y=y^x \Leftrightarrow [(dx_1)^{y_1})^d-[(dy_1)^{x_1}]^d=0 \Leftrightarrow ((dx_1)^{y_1}-(dy_1)^{x_1})B=0$

Vì $B>0$ nên $(dx_1)^{y_1}=(dy_1)^{x_1} \Rightarrow (x_1)^{y_1}=d^{x_1-y_1}(y_1)^{x_1} \Rightarrow y_1|x_1 \Rightarrow y_1=1 \Rightarrow y=d \Rightarrow x=ky$ 

Sau đó giải như trên, suy ra $ky=y^k$. 

Với $k=y=2 \Rightarrow x=4$

Với $k>2$ thì vế phải tăng nhanh hơn vế trái nên vô nghiệm 

Vậy nghiệm là $x=y$ hoặc $(x;y)=(2;4),(4;2)$




#643435 Topic tổng hợp các bài toán về phương trình nghiệm nguyên.

Đã gửi bởi Cantho2015 on 03-07-2016 - 13:27 trong Số học

Tìm nghiệm nguyên dương của phương trình: $a^2+b^2+c^2=2(ab+bc+ca)$

$a^2+b^2+c^2=2(ab+bc+ca) \Leftrightarrow (a-b)^2+(b-c)^2+(c-a)^2=a^2+b^2+c^2$

$\Rightarrow$ $\begin{cases} a-b=a\\c-a=a \end{cases}$ $\Rightarrow \begin{cases} a=0, b=c\\ a=0, b=c \end{cases}$

Hoặc $b-c=a \Rightarrow$ $\begin{cases} b-c=b\\c-a=b \end{cases}$ $\Rightarrow \begin{cases} c=0,b=a\\a=0, b=c \end{cases}$

Xét tương tự ta suy ra nghiệm pt là $a=0, b=c$ và $b=0, a=c$ và $c=0, b=a$




#640833 Topic tổng hợp các bài toán về phương trình nghiệm nguyên.

Đã gửi bởi Cantho2015 on 17-06-2016 - 11:46 trong Số học

Tìm số nguyên x để $2{{\text{x}}^{2}}-x-36$ là bình phương một số nguyên tố

$2x^2-x-36=(2x-9)(x+4)=p^2$ (p là số nguyên tố)

$\Rightarrow 2x-9=x+4$ hoặc $2x-9= \pm{1}$ hoặc $x+4= \pm{1}$

Giải ra được $(x;p)=(13;17),(5;3)$




#640977 Topic tổng hợp các bài toán về phương trình nghiệm nguyên.

Đã gửi bởi Cantho2015 on 18-06-2016 - 02:39 trong Số học

Tiếp nè:

     Tìm tất cả các cặp số nguyên không âm thoả mãn phương trình: $(y+1)^{4}+y^{4}=(x+1)^{2}+x^{2}$

 
 $$(y+1)^4+y^4=(x+1)^2+x^2$$
Giải:
 $\Leftrightarrow 2x^2+2x+1-(y+1)^4-y^4=0$
 $\Rightarrow \Delta_x=4-8(2y^4+4y^3+6y^2+4y)=k^2$ 
Vì vế trái luôn chia hết cho 2
 $\Rightarrow k=2m \Rightarrow 1-2(2y^4+4y^3+6y^2+4y)=m^2$
     Kẹp!
  Ta có:
 $ [1-2(2y^4+4y^3+6y^2+4y)]-(2y^2+2y+1)^2=4y^2+4y=4y(y+1) \geq{0}$, $\forall y \in \mathbb{Z}$ 
  Lại có:
 $ (2y^2+2y+2)^2 -[1-2(2y^4+4y^3+6y^2+4y)]=3 > 0$
 $ \Rightarrow(2y^2+2y+1)^2 \leq m^2 < (2y^2+2y+2)^2$
 $ \Rightarrow (2y^2+2y+1)^2=m^2 \Leftrightarrow 4y(y+1)=0 \Leftrightarrow y=0, y=-1$
 $ \Rightarrow (x;y)=(0;0),(-1;0),(-1;0),(-1;-1)$



#562536 ĐỀ THI VÒNG 1+VÒNG 2 MÔN TOÁN TUYỂN SINH VÀO LỚP $10$ THPT CHUYÊN...

Đã gửi bởi Cantho2015 on 30-05-2015 - 22:19 trong Tài liệu - Đề thi

'' | '' là chi vậy ạ? .___. :3


a | b là b chia hết cho a hay a là ước của b



#562525 ĐỀ THI VÒNG 1+VÒNG 2 MÔN TOÁN TUYỂN SINH VÀO LỚP $10$ THPT CHUYÊN...

Đã gửi bởi Cantho2015 on 30-05-2015 - 21:39 trong Tài liệu - Đề thi

Câu III
 
a) Dễ thấy $F\in AB$ và $E\in AC$


chứng minh được không chị



#562522 Đề thi 10 PTNK-DHQG 2015-2016 môn Toán (2 vòng)

Đã gửi bởi Cantho2015 on 30-05-2015 - 21:14 trong Tài liệu - Đề thi

5c) kéo dài OK cắt (O) tại J'. Ta có AJ'CD nội tiếp nên góc AJ'C=120 độ= 2 góc ABC, mà AJ=JC ( OJ là trung trực AC) suy ra J' trùng J.
Ta có góc AOJ= góc ADC=60 độ nên tam giác AOJ đều, dễ dàng tính được AK theo OJ mà AK=1/2AC=1/2.DE ( AECD là hình thang cân). Tính được OJ/DE

\dfrac {1}{\sqrt {3}}



#562514 Đề thi 10 PTNK-DHQG 2015-2016 môn Toán (2 vòng)

Đã gửi bởi Cantho2015 on 30-05-2015 - 20:56 trong Tài liệu - Đề thi

câu hình nhé, mấy câu còn lại @ rùi :v
$(a)$
$(b)$
Dễ thấy $MK$ là trung trực của $AC$. $\Rightarrow \measuredangle MAC=\measuredangle MCA=\measuredangle CDI$.
$(c)$
$M,K,I$ thẳng hàng. Do đó, chỉ cần tính:
$\frac{MK}{AC}=\frac{MK}{KC}$.
Mà: $\measuredangle COK=60^0$

Sao M,K,I thẳng hàng được bạn, với câu c đề kêu tính JO/DE mà



#562520 Đề thi 10 PTNK-DHQG 2015-2016 môn Toán (2 vòng)

Đã gửi bởi Cantho2015 on 30-05-2015 - 21:11 trong Tài liệu - Đề thi

5c) kéo dài OK cắt (O) tại J'. Ta có AJ'CD nội tiếp nên góc AJ'C=120 độ= 2 góc ABC, mà AJ=JC ( OJ là trung trực AC) suy ra J' trùng J.
Ta có góc AOJ= góc ADC=60 độ nên tam giác AOJ đều, dễ dàng tính được AK theo OJ mà AK=1/2AC=1/2.DE ( AECD là hình thang cân). Tính được OJ/DE



#563313 Đề thi tuyển sinh vào trường THPT chuyên Đại học Sư Phạm Hà Nội 2015-2016 (2...

Đã gửi bởi Cantho2015 on 03-06-2015 - 20:58 trong Tài liệu - Đề thi

bai 4

Hình gửi kèm

  • IMG_0160.JPG



#642107 Topic: Thảo luận về các bài tập trong chuyên đề số học của VMF.

Đã gửi bởi Cantho2015 on 25-06-2016 - 11:47 trong Số học

Việc làm rất ý nghĩa. T ủng hộ một bài:

 

b) Đặt $(m,n)=k$ ta có $m=m'k, n=n'k$ với $(m',n')=1$.

Khi đó $2^m-1=2^{m'k}-1=(2^k-1)P$. Tương tự $2^n-1=2^{n'k}-1=(2^k-1)Q$.

Chú ý $(m',n')=1$ nên $(P,Q)=1$ tức $(2^m-1; 2^n+1)=2^k-1=2^{(m,n)-1}$.

Cái chỗ $(m',n')=1 \Rightarrow (P,Q)=1$ em không hiểu lắm. Với em nghĩ $(2^m-1;2^n-1)=2^k-1$ chứ đâu phải $(2^m-1;2^n+1)=2^k-1$ 




#642104 Topic: Thảo luận về các bài tập trong chuyên đề số học của VMF.

Đã gửi bởi Cantho2015 on 25-06-2016 - 11:28 trong Số học

Bài 13:

Dãy Fermat phải là $$F_n=2^{2^n}+1$$ và có thể tổng quát lên là dãy $F_k=a^{2^k}+b^{2^k}$ với $gcd(a;b)=1$, $2|ab$ và $k \in \mathbb{Z^+}$

Giải: Vì $2|ab$ và $(a;b)=1$ nên giả sử $a$ là số chẵn thì $b$ phải là số lẻ. Giả sử $m>n$ với $m,n$ là các số tự nhiên bất kỳ. 
Gọi $(F_m;F_n)=d$ (d là số lẻ vì cả hai $F_m,F_n$ lẻ)
Ta có $a^{2^{n+1}}-b^{2^{n+1}}|a^{2^{n+1}2^{m-n-1}}-b^{2^{n+1}2^{m-n-1}}$ vì $(x-y|x^k-y^k)$ ở đây $(x=a^{2^{n+1}}, y=b^{2^{n+1}}, k=m-n-1)$
Mà $a^{2^{n}}+b^{2^{n}}|(a^{2^{n}})^2-(b^{2^{n}})^2$ suy ra $F_n|F_m-2b^{2^{m}}$
Từ $d|F_n$ và $d|F_m$ suy ra $d|2b^{2^{m}} \Rightarrow d|b^{2^{m}}$. Mà $d|F_m$ suy ra $d|a^{2^{m}}$ kết hợp với giả thuyết $(a;b)=1 \Rightarrow d=1$
(Với dãy Fermat $d|2 \Rightarrow d=1$ nhanh hơn tí)
 



#562541 Đề thi vào lớp 10 chuyên Thpt chuyên Nguyễn Du 2011-2012

Đã gửi bởi Cantho2015 on 30-05-2015 - 22:57 trong Tài liệu - Đề thi

Chứng minh H là trực tâm và cũng là trọng tâm của tam giác OAN đều
=>$HM=\frac{1}{3}AM$
Ta có $\widehat{QOB}=60^0$
=>Tam giác QOB đều
=>$\widehat{QBO}=60^0=\widehat{NOA}$ (do ON và QB song song do cùng vuông góc với AQ)
=>$\widehat{NOA}=\widehat{NOQ}=60^0$
Xét tam giác QOA có phân giác ON cũng là trung tuyến
=>$AM=AQ$
Lại có $3HM=AM$
=>$AQ=6HM$

góc QAB=60 độ thì sao tam giác OAN đều được khi đó góc AON= 30 độ mà



#562082 ĐỀ THI THỬ TUYỂN SINH VÀO LỚP 10 TRƯỜNG PTNK - ĐHQG TP.HCM

Đã gửi bởi Cantho2015 on 28-05-2015 - 12:32 trong Tài liệu - Đề thi

Mới chứng minh được câu a vs câu b.
a. $IM.IA=IF^2=IN.IB .. \Rightarrow AMNB$ nội tiếp.
c. Áp dụng Menelaus vào tam giác ABC có cát tuyến PFE thì $\frac{AF.FB.EC}{FB.PC.AE}=1 \rightarrow \frac{PB}{PC}=\frac{FB}{FC}=\frac{DB}{DC}$

Bạn dogsteven nói đúng rồi. Gọi K là giao điểm AD và (I) ta có FKED là tứ giác điều hoà (http://diendantoanho...attach_id=17128) là tứ giác tạo bởi hai tiếp tuyến AF,AE và cát tuyến AKD ấy ( tứ giác điều hoà có tích các cạnh đối bằng nhau). Đối với điểm P, kẻ PK' là tiếp tuyến của (I), lúc này PK',PD là tiếp tuyến, PEF là cát tuyến ta cũng có PK'ED là tứ giác điều hoà, suy ra FK'.DE=FD.K'E;FK.DE=FD.KE ( FKED tứ giác điều hoà). Tới đây dùng tam giác đồng dạng dễ dàng suy ra K' trùng K suy ra PK là tiếp tuyến của (I) suy ra PI vuông góc KD (t/c 2 tiếp tuyến cắt nhau) hay PI vuông góc AD (đpcm)



#563163 Đề toán thi vào 10 chuyên Vũng Tàu

Đã gửi bởi Cantho2015 on 03-06-2015 - 08:26 trong Tài liệu - Đề thi

Hình như chưa ai giải câu 1c 
$\left\{\begin{matrix} \sqrt{1+x}+\sqrt{1+y}=2\sqrt{2} & & \\ \sqrt{x}+\sqrt{y}=2 & & \end{matrix}\right.$
ĐKXĐ: $x,y \geq 0$
Ta có:
$(2\sqrt{2})^2=(x+y)+2\sqrt{(1+x)(y+1)}+2 \geq \frac{(\sqrt{x}+\sqrt{y})^2}{2}+2(\sqrt{x}+\sqrt{y})+2=8$
Do đó xảy ra đẳng thức $\Rightarrow \left\{\begin{matrix} x=y & & \\ xy=1 & & \end{matrix}\right.\rightarrow x=y=1$

bài này không bình phương pt (1) mà dùng bđt Minkowsky để đánh giá cũng được luôn



#643374 $3^{x}+4^{y}=5^{z}$

Đã gửi bởi Cantho2015 on 02-07-2016 - 22:28 trong Phương trình, hệ phương trình và bất phương trình

Giải phương trình:                      x= 4x

 

( Mình  mò được x = 2 và x = 4 nhưng không có cách nào để tóm được nó! Mọi người giúp với.) 

Xem bài toán tổng quát cho $x^y=y^x$ ở đây:

 

http://diendantoanho...-26#entry643358




#643380 $3^{x}+4^{y}=5^{z}$

Đã gửi bởi Cantho2015 on 02-07-2016 - 22:50 trong Phương trình, hệ phương trình và bất phương trình

Có ai hiểu tại sao $m=1$ không ạ?

Tới đây đặt $c=5^a, d=3^b$ $gcd(c,2)=1$

$5^{2a}-3^{2b}=(c-d)(c+d)=-2^{2y}$

Vì $c+d>c-d$ suy ra:

\begin{cases} c+d=2^n\\ c-d=-2^m\\ m+n=2y \end{cases}

suy ra$ 2c+2^m=2^n \tag{1}$ và $2^m=2^n+2d \tag{2}$ nếu $n \geq2$ thì ở pt $(2)$ suy ra $m \geq2$ ( vì $2d>0$). Vậy vế phải pt $(1)$ chia hết cho $4$ mà vế trái dư 2 ( vì $gcd(c,2)=1$) suy ra vô lý.

Vậy $m,n<2$ tới đây xét các trường hợp thì dễ rồi.

 

Ngoài ra còn có thể đánh giá $x,y,z \geq 3$ vô lý vì vế trái của $3^x+4^y \equiv 9+0 \equiv 1 \pmod{4}$ trong khi $5^z \equiv 5^3 \equiv 5 \pmod{3}$ vô lý

Sau đó xét các trường hợp $x,y,z<3$ 




#639998 Đề thi chuyên toán chuyên bắc ninh 2016

Đã gửi bởi Cantho2015 on 13-06-2016 - 09:50 trong Tài liệu - Đề thi

Bài 1

$P(x)=(x-1)(x^3+6x^2 +11x+6)=(x-1)(x+1)(x^2+5x+6)=(x+1)(x-1)(x+2)(x+3)$




#641184 Giải pt nghiệm nguyên: $x^2+y^2+xy=x^2y^2$

Đã gửi bởi Cantho2015 on 19-06-2016 - 04:36 trong Số học

Phương trình tương đương $(x+y)^2=xy(xy+1)$
Từ đây suy ra $(x;y)=(0;0),(1;-1),(-1;1)$.

Tại sao $(x+y)^2=xy(xy+1)$ thì suy ra được nghiệm vậy bạn?




#564814 Đề thi tuyển sinh lớp 10 chuyên toán THPT chuyên Lý Tự Trọng- Cần Thơ

Đã gửi bởi Cantho2015 on 10-06-2015 - 16:04 trong Tài liệu - Đề thi

IMG_0184.JPG




#641225 Chứng minh $x(x+1)=p^{2n}(y+1)$ vô nghiệm

Đã gửi bởi Cantho2015 on 19-06-2016 - 10:35 trong Số học

1. Chứng minh phương trình nghiệm nguyên $x(x+1)=p^{2n}y(y+1)$ vô nghiệm với $p$ là số nguyên tố và $n$ là số nguyên dương

2. Giải phương trình nghiệm nguyên với p là số nguyên tố, $x,y$ là hai số nguyên dương

$x^5+x^4+1=p^y$

3. Giải hệ phương trình nghiệm nguyên

$\begin{cases}x+y+z+u+v=xyuv+(x+y)(u+v)\\xy+z+uv=xy(u+v)+uv(x+y)\end{cases}$

 




#641344 Chứng minh $x(x+1)=p^{2n}(y+1)$ vô nghiệm

Đã gửi bởi Cantho2015 on 20-06-2016 - 01:49 trong Số học

Bài 1 mình giải thế này:

  $$\displaystyle{\frac{x(x+1)}{y(y+1)}=(p^n)^2}$$
 Dễ thấy \[gcd(x;x+1)=gcd(y;y+1)=1 \]
Giả sử $y|x \Rightarrow y \nmid x+1, y+1 \nmid x$
  $$x=my; x+1=n(y+1) $$ (với $m,n \geq{1}$)
  $(m;n)=d \Rightarrow d|my-n(y+1) \Rightarrow d|x-(x+1) \Rightarrow d=1$
  $mn=(p^n)^2=p.p.p....p$ $\Rightarrow$ Một trong $m$ hoặc $n$ phải bằng $1$ 
 Cả hai trường hợp $\Rightarrow x=y$. Vậy $p=1$ (vô lý)
  Nếu $y|x+1$, chứng minh tương tự



#641346 Chứng minh $x(x+1)=p^{2n}(y+1)$ vô nghiệm

Đã gửi bởi Cantho2015 on 20-06-2016 - 02:27 trong Số học

Bài 2 quan trọng là phát hiện:

$x^5+x^4+1=x^5+x^3+x^3+1-x^3=(x^2+x+1)(x^3-x+1)$

Viết pt lại: $(x^2+x+1)(x^3-x+1)=p^y$

Dễ thấy $x$ phải dương

Nếu $x=1$ thì $p=3,y=1$

Nếu $x>1$ thì khi đó $y>1$ (Do $x^2+x+1>1, x^3-x+1$)

Giả sử $(x^2+x+1,x^3-x+1)=d$

Suy ra $d | -(x^2+x+1+x^3-x+1)+x(x^2+x+1)=x-2$

$d| -(x-2)(x+3)+(x^2+x+1)=5$

Nên nếu $p=5$, thì $(x^2+x+1,x^3-x+1)=5$

 Mà $x^2+x+1 \ne 5$ và $x^3-x+1 \ne 5$ (với $x$ nguyên)

Nên $p=5$ phương trình vô nghiệm.

$p \ne 5$ thì suy ra: $(x^2+x+1,x^3-x+1)=1$ Mà $x^2+x+1 \ne 1$ và $x^3-x+1 \ne 1$ (với $x>1$ )

Vậy: $(x,y,p)=(1,1,3)$

Cái chỗ $(x^2+x+1,x^3-x+1)=5$ $\Rightarrow$ $x^2+x+1 \ne 5$ sao không phải là $x^2+x+1 \ne 5k$, mình không hiểu.